Statics Example: 2D Moments

Sdílet
Vložit
  • čas přidán 3. 09. 2015

Komentáře • 58

  • @abdalrahmanziady3742
    @abdalrahmanziady3742 Před 6 lety +50

    neeeegaat iam sorry positive

  • @jodifrye8877
    @jodifrye8877 Před 3 lety +2

    Thank you! This makes the components make more sense when talking about moments.

  • @emmastephens83
    @emmastephens83 Před 5 lety +13

    Something that I have come to think of when looking for the direction of moment in each component is thinking of each force as it moves along its own line of action, and then considering how I would have to 'steer' the arrow head in order to reach the point that I are trying to find the moment around. This has worked well for me in 2D problems, I am not sure how it would hold up in 3D problems.

  • @yoenespattyn6998
    @yoenespattyn6998 Před 6 lety +1

    Understand it clearly! Thanks man!

  • @karimkhan1312
    @karimkhan1312 Před 7 lety

    sir - you are the best on this topic

  • @cappecat_
    @cappecat_ Před 5 lety +14

    lol i was looking for 2D from the Gorillaz I clicked because I thought 2D was teaching Statics XD

  • @emilianojimenez1075
    @emilianojimenez1075 Před 2 lety +3

    how did you get cosine and sine for the first 2 forces?

  • @wallwall9172
    @wallwall9172 Před 5 lety +2

    Better than my teacher... 😂Thank you

  • @TheRedAirOn
    @TheRedAirOn Před rokem +1

    Nice video, Mark Holdhusen.

  • @karimkhan1312
    @karimkhan1312 Před 8 lety

    thank u sir for excelent presentation

  • @KingCobra2105
    @KingCobra2105 Před 9 měsíci

    thanks for your effort fully understand the Moment

  • @carlosombao1767
    @carlosombao1767 Před 5 lety +1

    Don't know if you're still active but, why is the (500 x 3/4) force taken as 4 meteres. I thought a moment was M=Fd(perpedicular) where d is distance from pivot point A?

  • @RozArialind
    @RozArialind Před 8 lety +9

    A number of questions, if I may.
    1) Why didn't you break the first two into the vertial and horizontal?
    2) Why is it -5(500 4/5)?

    • @BLdontM
      @BLdontM Před 8 lety +8

      +Rozalind Aria Since the horizontal component of the first two line up perfectly with point A... as in they pass through it, there is no moment created by that component of the force. So you could break them up into components but you'd find the horizontal components come out to 0. The second question: The triangle is used to resolve that force into components. If you're trying to find the Y component, the leg of the triangle parallel to the Y axis is used. Sin = y/r cos = x/r Same thing with the X component of that force using the triangle. The part that is parallel is 3, so the X component of that force would be 3/5 x 500.

    • @trexinvert
      @trexinvert Před 7 lety +3

      Did you understand the answer by Rusty. Another smart guy who is thinking 2 steps ahead of us.
      1.) Your question is valid. You can break the first forces(F1 and F2) into it's component. When you do that you will get F1(horiz) x 0 meters = 0 and F2(horiz) x 0 meters = 0. So basically, do the extra work to write it out completely and you can't. Later it becomes instinctive.
      2.) Work out the trigonometry of F3 directions. In fact, find the angle between the 4 and the 5 legs. What do you get? Then review your trigonometry again. You know opp/hyp and adj/hyp. What do you see. You will see that just dividing by the legs "is in fact" the same thing as finding the angle and then using sin and cosines. Except, you do 1 or 2 steps more. You see dividing by the legs(if you have all 3 defined) is exactly the same thing as if you had the angle. In some cases, it's faster and easier to check back your written arithmetic.

  • @shahidAhmad9430
    @shahidAhmad9430 Před 6 lety +3

    A wonderful video, sir if you please upload some videos of dynamics , i will be very thankful

  • @chaz3345
    @chaz3345 Před 3 lety +1

    So you use the X plane distance in the moment multiplication for the y force and the Y plane distance from the point for the multiplication of the force for the X direction. It's confusing.

  • @malcolmmukutu5214
    @malcolmmukutu5214 Před 7 lety +1

    well explained thank you man :)

  • @elegance1996
    @elegance1996 Před 5 lety +5

    how does a force going to the right produce a CCW rotation about a?

  • @mznaahmed6826
    @mznaahmed6826 Před 2 lety

    THANK YOU!

  • @abdulkabiryahya8822
    @abdulkabiryahya8822 Před 3 lety

    Why are the 2 and 3 negative?

  • @kaeli5445
    @kaeli5445 Před rokem

    in moment_A1 it should be a 250 sin 30
    because its respect to y axis

  • @DeadMau2012
    @DeadMau2012 Před 4 lety +8

    COMponent

  • @solo451
    @solo451 Před rokem +1

    i thought Fy should be sin not cos?

  • @bolisa.k4254
    @bolisa.k4254 Před rokem

    I thought counter clockwise is the same as anticlockwise

  • @sihlegidzana
    @sihlegidzana Před 5 lety

    thanks post more videos please

  • @jcflores1774
    @jcflores1774 Před 6 lety +4

    why is the horizontal 4/5 shouldn't it be 3/5? and the vertical 4/5

    • @jacobj2842
      @jacobj2842 Před 3 lety

      Because 4/5 is perpendicular to the horizontal distance. 😁

  • @UppatownNyc
    @UppatownNyc Před 8 lety +14

    Why are you using cosine for the first one?

    • @dcwood2137
      @dcwood2137 Před 4 lety +4

      The angle is formed at the vertical instead of the horizontal. So, the force in the y direction is a measurement of the side adjacent to the angle.

    • @Chloholio
      @Chloholio Před 3 lety +2

      alternatively you can use -2 x 250sin120 if that helps you visualize it better. a lot of physics people use trig angle identities from the unit circle and it confuses the hell out of people (including myself)

  • @mauriciorv228
    @mauriciorv228 Před 4 lety

    why is Fx component 4 meters? i don´t quite get that

    • @ricardjunroloma2307
      @ricardjunroloma2307 Před 3 lety

      Force × its perpendicular distance . For Fx, since it is a horizontal force its perpendicular distance from point A should be the vertical length which is 4m in the given example.

  • @craig7878
    @craig7878 Před 11 měsíci

    Should the y component of F1 be 250sin(30) ?

    • @elifnur9186
      @elifnur9186 Před 3 měsíci

      nope because in the first one, Fy should be 250×cos30 according to the Pythagoras

  • @dextergeeunabia2262
    @dextergeeunabia2262 Před 9 měsíci

    I am really confused. Who were telling you the truth? On the other video I watched, he said that the counter clockwise was the negative and the clockwise was the positive.

  • @itsnotrhenlopez7626
    @itsnotrhenlopez7626 Před 6 lety +2

    clockwise should be positive?

    • @blackconfucius4974
      @blackconfucius4974 Před 5 lety +3

      Jhen Dela Rosa no

    • @roserizal5499
      @roserizal5499 Před 4 lety +1

      yes, the vid was wrong

    • @Witness1037
      @Witness1037 Před 4 lety +3

      Clockwise is negative, counter clockwise is positive. Seems backwards but that’s the way it is

  • @GabrielAdda-y1u
    @GabrielAdda-y1u Před 6 dny

    I got a different answer 2349.05Nm

  • @luiscorona552
    @luiscorona552 Před 7 lety +2

    Why is the first one not sin 30?????

    • @victorlobatodasilvacosta9848
      @victorlobatodasilvacosta9848 Před 7 lety +5

      If you take sin30, you will be finding the Fx which is not perpendicular to the distance from point A.

    • @trexinvert
      @trexinvert Před 7 lety +4

      Did you understand Victor's answer above? He's obviously thinking two steps ahead of us.
      Basically, let's consider "both components". F1(x-dir) and F1(y-dir). The moment at A of force F1 is the following:
      F1(y-dir) x 2 meters= F1(cos30) x 2meters
      F1(x-dir) x 0 meters = F1(sin30) x 0 meters = 0
      I know for some students it's a no brainer, but I can understand people getting confused by the rules/conventions.
      You must be absolutely 100% certain/confident of the rules to apply vector statics. If you try to mix some logic you will quickly get lost

    • @luiscorona552
      @luiscorona552 Před 7 lety

      trexinvert yea I agree I approached in a logical manner I remember this problem haha

    • @potatoguy8390
      @potatoguy8390 Před 7 lety +2

      you can use sin but you will use 60 instead of 30.

    • @meerycleardevenosa8050
      @meerycleardevenosa8050 Před 7 lety

      haha

  • @johnmifsud6814
    @johnmifsud6814 Před 2 lety

    cos 30 is cos 30 radians - silly not writing the degree symbol. Feeble students make this mistake.

    • @solo451
      @solo451 Před rokem

      but you get the same answer with or without it when you pput it in calculator

  • @UppatownNyc
    @UppatownNyc Před 8 lety

    Why are you using cosine for the first one?

    • @abuhassanabo
      @abuhassanabo Před 8 lety

      Because that creates the vertical force on the body. The horisontal force (sin 30) goes right theough A and is therefore 0.